Rudin exercise 3.8 solutionShowing a series converges absolutely iff it has a uniform boundIf $sum a_n$ is convergent, with $a_n geq 0$ $forall n in mathbbN$, show that if $p > 1$, $sum a_n^p$ converges.Why doesn't this work for Rudin Exercise 3.8Convergence of recursive sequence $a_n+1 =frac 1k left(a_n + fracka_nright)$Two p-norms are not equivalent for different p on $ell_1$Sum of squares converges given sum doesWhat's significant in Abel's Theorem Proof in Baby Rudin?Why is my proof wrong? (Rudin Ch 3 #8) If $sum a_n$ converges, and if $b_n$ is monotonic and bounded, prove that $sum a_n b_n$ converges.Calculus-probabilityProof Verification: A monotonically increasing sequence that is bounded above always has a LUB

Is it safe to use olive oil to clean the ear wax?

Strong empirical falsification of quantum mechanics based on vacuum energy density

Why did the EU agree to delay the Brexit deadline?

Redundant comparison & "if" before assignment

How should I respond when I lied about my education and the company finds out through background check?

What should you do when eye contact makes your subordinate uncomfortable?

What is the evidence for the "tyranny of the majority problem" in a direct democracy context?

What prevents the use of a multi-segment ILS for non-straight approaches?

Fear of getting stuck on one programming language / technology that is not used in my country

Why should universal income be universal?

I am looking for the correct translation of love for the phrase "in this sign love"

Problem with TransformedDistribution

Non-trope happy ending?

Pre-mixing cryogenic fuels and using only one fuel tank

Argument list too long when zipping large list of certain files in a folder

why `nmap 192.168.1.97` returns less services than `nmap 127.0.0.1`?

Is it possible to have a strip of cold climate in the middle of a planet?

How to implement a feedback to keep the DC gain at zero for this conceptual passive filter?

Why is so much work done on numerical verification of the Riemann Hypothesis?

"Spoil" vs "Ruin"

How to bake one texture for one mesh with multiple textures blender 2.8

Can I sign legal documents with a smiley face?

Freedom of speech and where it applies

Creepy dinosaur pc game identification



Rudin exercise 3.8 solution


Showing a series converges absolutely iff it has a uniform boundIf $sum a_n$ is convergent, with $a_n geq 0$ $forall n in mathbbN$, show that if $p > 1$, $sum a_n^p$ converges.Why doesn't this work for Rudin Exercise 3.8Convergence of recursive sequence $a_n+1 =frac 1k left(a_n + fracka_nright)$Two p-norms are not equivalent for different p on $ell_1$Sum of squares converges given sum doesWhat's significant in Abel's Theorem Proof in Baby Rudin?Why is my proof wrong? (Rudin Ch 3 #8) If $sum a_n$ converges, and if $b_n$ is monotonic and bounded, prove that $sum a_n b_n$ converges.Calculus-probabilityProof Verification: A monotonically increasing sequence that is bounded above always has a LUB













0












$begingroup$


Problem . if the sequence $sumlimits_a_n$ converges and $b_n$ is a monotonic bounded sequence then $sumlimits_ a_nb_n$ converges.



Solution. Let's use the identity:
$sumlimits_n=p^q a_nb_n$ = $sumlimits_n=p^q-1 s_n(b_n-b_n-1)+s_qb_q-s_p-1b_p$, where $s_n=sumlimits_k=1^n a_n$. Then $|sumlimits_n=p^q-1 s_n(b_n-b_n-1)|<=|sumlimits_n=p^q-1 s_n|B$, where B is the difference of upper and lower bound of the sequence $b_n$. since $a_n$ is convergent we can make $|sumlimits_n=p^q-1 s_n|$ as small as we want and therefore we can make this $|sumlimits_n=p^q-1 s_n(b_n-b_n-1)|$ as small as we want like $x/3$. Then let's take the absolute value of another term of the right side of the identity.
$|s_qb_q|<=|s_q||b_up|$, where $b_up$ is the upper bound of the sequence and the same argument here too. we can make this term as small as we want. The same goes to the absolute value of the third term on the right side of the identity. Finally $|sumlimits_n=p^q a_nb_n|<=|sumlimits_n=p^q-1 s_n(b_n-b_n-1)|+|s_qb_q|+|s_p-1b_p|$. We can find $p$ and $q$ to make the right side small enough, completing the proof. Is this right? I am not satisfied what I have written here, I should have stated better in my opinion but the general idea of the proof is shown I suppose.










share|cite|improve this question











$endgroup$











  • $begingroup$
    yeah, "making as small as we want" is a bad idea, but I can say that I can bound that sum by taking appropriate $p$ and $q$.
    $endgroup$
    – shota kobakhidze
    Mar 15 at 18:28















0












$begingroup$


Problem . if the sequence $sumlimits_a_n$ converges and $b_n$ is a monotonic bounded sequence then $sumlimits_ a_nb_n$ converges.



Solution. Let's use the identity:
$sumlimits_n=p^q a_nb_n$ = $sumlimits_n=p^q-1 s_n(b_n-b_n-1)+s_qb_q-s_p-1b_p$, where $s_n=sumlimits_k=1^n a_n$. Then $|sumlimits_n=p^q-1 s_n(b_n-b_n-1)|<=|sumlimits_n=p^q-1 s_n|B$, where B is the difference of upper and lower bound of the sequence $b_n$. since $a_n$ is convergent we can make $|sumlimits_n=p^q-1 s_n|$ as small as we want and therefore we can make this $|sumlimits_n=p^q-1 s_n(b_n-b_n-1)|$ as small as we want like $x/3$. Then let's take the absolute value of another term of the right side of the identity.
$|s_qb_q|<=|s_q||b_up|$, where $b_up$ is the upper bound of the sequence and the same argument here too. we can make this term as small as we want. The same goes to the absolute value of the third term on the right side of the identity. Finally $|sumlimits_n=p^q a_nb_n|<=|sumlimits_n=p^q-1 s_n(b_n-b_n-1)|+|s_qb_q|+|s_p-1b_p|$. We can find $p$ and $q$ to make the right side small enough, completing the proof. Is this right? I am not satisfied what I have written here, I should have stated better in my opinion but the general idea of the proof is shown I suppose.










share|cite|improve this question











$endgroup$











  • $begingroup$
    yeah, "making as small as we want" is a bad idea, but I can say that I can bound that sum by taking appropriate $p$ and $q$.
    $endgroup$
    – shota kobakhidze
    Mar 15 at 18:28













0












0








0





$begingroup$


Problem . if the sequence $sumlimits_a_n$ converges and $b_n$ is a monotonic bounded sequence then $sumlimits_ a_nb_n$ converges.



Solution. Let's use the identity:
$sumlimits_n=p^q a_nb_n$ = $sumlimits_n=p^q-1 s_n(b_n-b_n-1)+s_qb_q-s_p-1b_p$, where $s_n=sumlimits_k=1^n a_n$. Then $|sumlimits_n=p^q-1 s_n(b_n-b_n-1)|<=|sumlimits_n=p^q-1 s_n|B$, where B is the difference of upper and lower bound of the sequence $b_n$. since $a_n$ is convergent we can make $|sumlimits_n=p^q-1 s_n|$ as small as we want and therefore we can make this $|sumlimits_n=p^q-1 s_n(b_n-b_n-1)|$ as small as we want like $x/3$. Then let's take the absolute value of another term of the right side of the identity.
$|s_qb_q|<=|s_q||b_up|$, where $b_up$ is the upper bound of the sequence and the same argument here too. we can make this term as small as we want. The same goes to the absolute value of the third term on the right side of the identity. Finally $|sumlimits_n=p^q a_nb_n|<=|sumlimits_n=p^q-1 s_n(b_n-b_n-1)|+|s_qb_q|+|s_p-1b_p|$. We can find $p$ and $q$ to make the right side small enough, completing the proof. Is this right? I am not satisfied what I have written here, I should have stated better in my opinion but the general idea of the proof is shown I suppose.










share|cite|improve this question











$endgroup$




Problem . if the sequence $sumlimits_a_n$ converges and $b_n$ is a monotonic bounded sequence then $sumlimits_ a_nb_n$ converges.



Solution. Let's use the identity:
$sumlimits_n=p^q a_nb_n$ = $sumlimits_n=p^q-1 s_n(b_n-b_n-1)+s_qb_q-s_p-1b_p$, where $s_n=sumlimits_k=1^n a_n$. Then $|sumlimits_n=p^q-1 s_n(b_n-b_n-1)|<=|sumlimits_n=p^q-1 s_n|B$, where B is the difference of upper and lower bound of the sequence $b_n$. since $a_n$ is convergent we can make $|sumlimits_n=p^q-1 s_n|$ as small as we want and therefore we can make this $|sumlimits_n=p^q-1 s_n(b_n-b_n-1)|$ as small as we want like $x/3$. Then let's take the absolute value of another term of the right side of the identity.
$|s_qb_q|<=|s_q||b_up|$, where $b_up$ is the upper bound of the sequence and the same argument here too. we can make this term as small as we want. The same goes to the absolute value of the third term on the right side of the identity. Finally $|sumlimits_n=p^q a_nb_n|<=|sumlimits_n=p^q-1 s_n(b_n-b_n-1)|+|s_qb_q|+|s_p-1b_p|$. We can find $p$ and $q$ to make the right side small enough, completing the proof. Is this right? I am not satisfied what I have written here, I should have stated better in my opinion but the general idea of the proof is shown I suppose.







real-analysis proof-verification






share|cite|improve this question















share|cite|improve this question













share|cite|improve this question




share|cite|improve this question








edited Mar 15 at 18:05









Alex Provost

15.6k22351




15.6k22351










asked Mar 15 at 17:59









shota kobakhidzeshota kobakhidze

115




115











  • $begingroup$
    yeah, "making as small as we want" is a bad idea, but I can say that I can bound that sum by taking appropriate $p$ and $q$.
    $endgroup$
    – shota kobakhidze
    Mar 15 at 18:28
















  • $begingroup$
    yeah, "making as small as we want" is a bad idea, but I can say that I can bound that sum by taking appropriate $p$ and $q$.
    $endgroup$
    – shota kobakhidze
    Mar 15 at 18:28















$begingroup$
yeah, "making as small as we want" is a bad idea, but I can say that I can bound that sum by taking appropriate $p$ and $q$.
$endgroup$
– shota kobakhidze
Mar 15 at 18:28




$begingroup$
yeah, "making as small as we want" is a bad idea, but I can say that I can bound that sum by taking appropriate $p$ and $q$.
$endgroup$
– shota kobakhidze
Mar 15 at 18:28










2 Answers
2






active

oldest

votes


















2












$begingroup$

Assume that $b_n$ is increasing. If $s_n=sum_k=1^n a_k$, then
$$
sum_k=1^n a_kb_k=sum_k=1^n (s_k-s_k-1)b_k=sum_k=1^n s_kb_k-sum_k=2^n s_k-1b_k=s_nb_n+
sum_k=2^ns_k-1(b_k-1-b_k).
$$

Now $s_nb_n$ is convergent, as a product of convergent sequences, while the series
$sum s_n-1(b_n-1-b_n)$
is dominated by the series $sum M(b_n-b_n-1)$, where $M=sup_ninmathbb N|s_n|$, and since $b_n-b_n-1$ is absolutely summable,
so is $sum s_n-1(b_n-1-b_n)$.






share|cite|improve this answer









$endgroup$












  • $begingroup$
    yeah, it looks like a similar idea, stated brilliantly.
    $endgroup$
    – shota kobakhidze
    Mar 15 at 19:11


















1












$begingroup$

The general idea is correct. The expression “the upper bound of the sequence” is not appropriate (there are infinitely many upper bounds). I think that it would be better to say that $b_up$ is an upper bound of the sequence $bigl(lvert b_nrvertbigr)_ninmathbb N$. With this choice, $b_up$ is non-negative. Also, with this choice you have$$leftlvertsum_n=p^q-1s_n(b_n-b_n-1)rightrvertleqslant2leftlvertsum_n=p^q-1s_nrightrvert b_up.$$Again, the expression “the difference of upper and lower bound of the sequence $b_n$” is not appropriate.






share|cite|improve this answer











$endgroup$












  • $begingroup$
    yeah, understood. Thanks for the response. By the way, saying that "we can make it as small as we want" is incorrect right? It would be more correct to say "bounded". Then the whole sum becomes "bounded" and not "as small as I wanted".
    $endgroup$
    – shota kobakhidze
    Mar 15 at 18:38










  • $begingroup$
    Oops! You are right. What you shoud use at that point is that $(s_n)_ninmathbb N$ is a boounded sequence and that the series $sum_n=1^inftylvert b_n-b_n+1rvert$ converges (this follows from the fact that $(b_n)_ninmathbb N$ is a monotonic and bounded sequence).
    $endgroup$
    – José Carlos Santos
    Mar 15 at 18:47











  • $begingroup$
    By the way, is it necessary for $b_n$ to be monotonic? cause I haven't used that fact, I just used the boundedness.
    $endgroup$
    – shota kobakhidze
    Mar 15 at 23:16










  • $begingroup$
    See my previous comment. Without that sequence being monotonic, how do you know that the series $sum_n=1^inftylvert b_n-b_n+1rvert$ converges?
    $endgroup$
    – José Carlos Santos
    Mar 15 at 23:33










  • $begingroup$
    yes, I see. It was my bad. Thanks :)
    $endgroup$
    – shota kobakhidze
    Mar 16 at 0:00










Your Answer





StackExchange.ifUsing("editor", function ()
return StackExchange.using("mathjaxEditing", function ()
StackExchange.MarkdownEditor.creationCallbacks.add(function (editor, postfix)
StackExchange.mathjaxEditing.prepareWmdForMathJax(editor, postfix, [["$", "$"], ["\\(","\\)"]]);
);
);
, "mathjax-editing");

StackExchange.ready(function()
var channelOptions =
tags: "".split(" "),
id: "69"
;
initTagRenderer("".split(" "), "".split(" "), channelOptions);

StackExchange.using("externalEditor", function()
// Have to fire editor after snippets, if snippets enabled
if (StackExchange.settings.snippets.snippetsEnabled)
StackExchange.using("snippets", function()
createEditor();
);

else
createEditor();

);

function createEditor()
StackExchange.prepareEditor(
heartbeatType: 'answer',
autoActivateHeartbeat: false,
convertImagesToLinks: true,
noModals: true,
showLowRepImageUploadWarning: true,
reputationToPostImages: 10,
bindNavPrevention: true,
postfix: "",
imageUploader:
brandingHtml: "Powered by u003ca class="icon-imgur-white" href="https://imgur.com/"u003eu003c/au003e",
contentPolicyHtml: "User contributions licensed under u003ca href="https://creativecommons.org/licenses/by-sa/3.0/"u003ecc by-sa 3.0 with attribution requiredu003c/au003e u003ca href="https://stackoverflow.com/legal/content-policy"u003e(content policy)u003c/au003e",
allowUrls: true
,
noCode: true, onDemand: true,
discardSelector: ".discard-answer"
,immediatelyShowMarkdownHelp:true
);



);













draft saved

draft discarded


















StackExchange.ready(
function ()
StackExchange.openid.initPostLogin('.new-post-login', 'https%3a%2f%2fmath.stackexchange.com%2fquestions%2f3149615%2frudin-exercise-3-8-solution%23new-answer', 'question_page');

);

Post as a guest















Required, but never shown

























2 Answers
2






active

oldest

votes








2 Answers
2






active

oldest

votes









active

oldest

votes






active

oldest

votes









2












$begingroup$

Assume that $b_n$ is increasing. If $s_n=sum_k=1^n a_k$, then
$$
sum_k=1^n a_kb_k=sum_k=1^n (s_k-s_k-1)b_k=sum_k=1^n s_kb_k-sum_k=2^n s_k-1b_k=s_nb_n+
sum_k=2^ns_k-1(b_k-1-b_k).
$$

Now $s_nb_n$ is convergent, as a product of convergent sequences, while the series
$sum s_n-1(b_n-1-b_n)$
is dominated by the series $sum M(b_n-b_n-1)$, where $M=sup_ninmathbb N|s_n|$, and since $b_n-b_n-1$ is absolutely summable,
so is $sum s_n-1(b_n-1-b_n)$.






share|cite|improve this answer









$endgroup$












  • $begingroup$
    yeah, it looks like a similar idea, stated brilliantly.
    $endgroup$
    – shota kobakhidze
    Mar 15 at 19:11















2












$begingroup$

Assume that $b_n$ is increasing. If $s_n=sum_k=1^n a_k$, then
$$
sum_k=1^n a_kb_k=sum_k=1^n (s_k-s_k-1)b_k=sum_k=1^n s_kb_k-sum_k=2^n s_k-1b_k=s_nb_n+
sum_k=2^ns_k-1(b_k-1-b_k).
$$

Now $s_nb_n$ is convergent, as a product of convergent sequences, while the series
$sum s_n-1(b_n-1-b_n)$
is dominated by the series $sum M(b_n-b_n-1)$, where $M=sup_ninmathbb N|s_n|$, and since $b_n-b_n-1$ is absolutely summable,
so is $sum s_n-1(b_n-1-b_n)$.






share|cite|improve this answer









$endgroup$












  • $begingroup$
    yeah, it looks like a similar idea, stated brilliantly.
    $endgroup$
    – shota kobakhidze
    Mar 15 at 19:11













2












2








2





$begingroup$

Assume that $b_n$ is increasing. If $s_n=sum_k=1^n a_k$, then
$$
sum_k=1^n a_kb_k=sum_k=1^n (s_k-s_k-1)b_k=sum_k=1^n s_kb_k-sum_k=2^n s_k-1b_k=s_nb_n+
sum_k=2^ns_k-1(b_k-1-b_k).
$$

Now $s_nb_n$ is convergent, as a product of convergent sequences, while the series
$sum s_n-1(b_n-1-b_n)$
is dominated by the series $sum M(b_n-b_n-1)$, where $M=sup_ninmathbb N|s_n|$, and since $b_n-b_n-1$ is absolutely summable,
so is $sum s_n-1(b_n-1-b_n)$.






share|cite|improve this answer









$endgroup$



Assume that $b_n$ is increasing. If $s_n=sum_k=1^n a_k$, then
$$
sum_k=1^n a_kb_k=sum_k=1^n (s_k-s_k-1)b_k=sum_k=1^n s_kb_k-sum_k=2^n s_k-1b_k=s_nb_n+
sum_k=2^ns_k-1(b_k-1-b_k).
$$

Now $s_nb_n$ is convergent, as a product of convergent sequences, while the series
$sum s_n-1(b_n-1-b_n)$
is dominated by the series $sum M(b_n-b_n-1)$, where $M=sup_ninmathbb N|s_n|$, and since $b_n-b_n-1$ is absolutely summable,
so is $sum s_n-1(b_n-1-b_n)$.







share|cite|improve this answer












share|cite|improve this answer



share|cite|improve this answer










answered Mar 15 at 19:03









Yiorgos S. SmyrlisYiorgos S. Smyrlis

63.6k1385165




63.6k1385165











  • $begingroup$
    yeah, it looks like a similar idea, stated brilliantly.
    $endgroup$
    – shota kobakhidze
    Mar 15 at 19:11
















  • $begingroup$
    yeah, it looks like a similar idea, stated brilliantly.
    $endgroup$
    – shota kobakhidze
    Mar 15 at 19:11















$begingroup$
yeah, it looks like a similar idea, stated brilliantly.
$endgroup$
– shota kobakhidze
Mar 15 at 19:11




$begingroup$
yeah, it looks like a similar idea, stated brilliantly.
$endgroup$
– shota kobakhidze
Mar 15 at 19:11











1












$begingroup$

The general idea is correct. The expression “the upper bound of the sequence” is not appropriate (there are infinitely many upper bounds). I think that it would be better to say that $b_up$ is an upper bound of the sequence $bigl(lvert b_nrvertbigr)_ninmathbb N$. With this choice, $b_up$ is non-negative. Also, with this choice you have$$leftlvertsum_n=p^q-1s_n(b_n-b_n-1)rightrvertleqslant2leftlvertsum_n=p^q-1s_nrightrvert b_up.$$Again, the expression “the difference of upper and lower bound of the sequence $b_n$” is not appropriate.






share|cite|improve this answer











$endgroup$












  • $begingroup$
    yeah, understood. Thanks for the response. By the way, saying that "we can make it as small as we want" is incorrect right? It would be more correct to say "bounded". Then the whole sum becomes "bounded" and not "as small as I wanted".
    $endgroup$
    – shota kobakhidze
    Mar 15 at 18:38










  • $begingroup$
    Oops! You are right. What you shoud use at that point is that $(s_n)_ninmathbb N$ is a boounded sequence and that the series $sum_n=1^inftylvert b_n-b_n+1rvert$ converges (this follows from the fact that $(b_n)_ninmathbb N$ is a monotonic and bounded sequence).
    $endgroup$
    – José Carlos Santos
    Mar 15 at 18:47











  • $begingroup$
    By the way, is it necessary for $b_n$ to be monotonic? cause I haven't used that fact, I just used the boundedness.
    $endgroup$
    – shota kobakhidze
    Mar 15 at 23:16










  • $begingroup$
    See my previous comment. Without that sequence being monotonic, how do you know that the series $sum_n=1^inftylvert b_n-b_n+1rvert$ converges?
    $endgroup$
    – José Carlos Santos
    Mar 15 at 23:33










  • $begingroup$
    yes, I see. It was my bad. Thanks :)
    $endgroup$
    – shota kobakhidze
    Mar 16 at 0:00















1












$begingroup$

The general idea is correct. The expression “the upper bound of the sequence” is not appropriate (there are infinitely many upper bounds). I think that it would be better to say that $b_up$ is an upper bound of the sequence $bigl(lvert b_nrvertbigr)_ninmathbb N$. With this choice, $b_up$ is non-negative. Also, with this choice you have$$leftlvertsum_n=p^q-1s_n(b_n-b_n-1)rightrvertleqslant2leftlvertsum_n=p^q-1s_nrightrvert b_up.$$Again, the expression “the difference of upper and lower bound of the sequence $b_n$” is not appropriate.






share|cite|improve this answer











$endgroup$












  • $begingroup$
    yeah, understood. Thanks for the response. By the way, saying that "we can make it as small as we want" is incorrect right? It would be more correct to say "bounded". Then the whole sum becomes "bounded" and not "as small as I wanted".
    $endgroup$
    – shota kobakhidze
    Mar 15 at 18:38










  • $begingroup$
    Oops! You are right. What you shoud use at that point is that $(s_n)_ninmathbb N$ is a boounded sequence and that the series $sum_n=1^inftylvert b_n-b_n+1rvert$ converges (this follows from the fact that $(b_n)_ninmathbb N$ is a monotonic and bounded sequence).
    $endgroup$
    – José Carlos Santos
    Mar 15 at 18:47











  • $begingroup$
    By the way, is it necessary for $b_n$ to be monotonic? cause I haven't used that fact, I just used the boundedness.
    $endgroup$
    – shota kobakhidze
    Mar 15 at 23:16










  • $begingroup$
    See my previous comment. Without that sequence being monotonic, how do you know that the series $sum_n=1^inftylvert b_n-b_n+1rvert$ converges?
    $endgroup$
    – José Carlos Santos
    Mar 15 at 23:33










  • $begingroup$
    yes, I see. It was my bad. Thanks :)
    $endgroup$
    – shota kobakhidze
    Mar 16 at 0:00













1












1








1





$begingroup$

The general idea is correct. The expression “the upper bound of the sequence” is not appropriate (there are infinitely many upper bounds). I think that it would be better to say that $b_up$ is an upper bound of the sequence $bigl(lvert b_nrvertbigr)_ninmathbb N$. With this choice, $b_up$ is non-negative. Also, with this choice you have$$leftlvertsum_n=p^q-1s_n(b_n-b_n-1)rightrvertleqslant2leftlvertsum_n=p^q-1s_nrightrvert b_up.$$Again, the expression “the difference of upper and lower bound of the sequence $b_n$” is not appropriate.






share|cite|improve this answer











$endgroup$



The general idea is correct. The expression “the upper bound of the sequence” is not appropriate (there are infinitely many upper bounds). I think that it would be better to say that $b_up$ is an upper bound of the sequence $bigl(lvert b_nrvertbigr)_ninmathbb N$. With this choice, $b_up$ is non-negative. Also, with this choice you have$$leftlvertsum_n=p^q-1s_n(b_n-b_n-1)rightrvertleqslant2leftlvertsum_n=p^q-1s_nrightrvert b_up.$$Again, the expression “the difference of upper and lower bound of the sequence $b_n$” is not appropriate.







share|cite|improve this answer














share|cite|improve this answer



share|cite|improve this answer








edited Mar 15 at 19:16









J. W. Tanner

3,6631320




3,6631320










answered Mar 15 at 18:29









José Carlos SantosJosé Carlos Santos

169k23132238




169k23132238











  • $begingroup$
    yeah, understood. Thanks for the response. By the way, saying that "we can make it as small as we want" is incorrect right? It would be more correct to say "bounded". Then the whole sum becomes "bounded" and not "as small as I wanted".
    $endgroup$
    – shota kobakhidze
    Mar 15 at 18:38










  • $begingroup$
    Oops! You are right. What you shoud use at that point is that $(s_n)_ninmathbb N$ is a boounded sequence and that the series $sum_n=1^inftylvert b_n-b_n+1rvert$ converges (this follows from the fact that $(b_n)_ninmathbb N$ is a monotonic and bounded sequence).
    $endgroup$
    – José Carlos Santos
    Mar 15 at 18:47











  • $begingroup$
    By the way, is it necessary for $b_n$ to be monotonic? cause I haven't used that fact, I just used the boundedness.
    $endgroup$
    – shota kobakhidze
    Mar 15 at 23:16










  • $begingroup$
    See my previous comment. Without that sequence being monotonic, how do you know that the series $sum_n=1^inftylvert b_n-b_n+1rvert$ converges?
    $endgroup$
    – José Carlos Santos
    Mar 15 at 23:33










  • $begingroup$
    yes, I see. It was my bad. Thanks :)
    $endgroup$
    – shota kobakhidze
    Mar 16 at 0:00
















  • $begingroup$
    yeah, understood. Thanks for the response. By the way, saying that "we can make it as small as we want" is incorrect right? It would be more correct to say "bounded". Then the whole sum becomes "bounded" and not "as small as I wanted".
    $endgroup$
    – shota kobakhidze
    Mar 15 at 18:38










  • $begingroup$
    Oops! You are right. What you shoud use at that point is that $(s_n)_ninmathbb N$ is a boounded sequence and that the series $sum_n=1^inftylvert b_n-b_n+1rvert$ converges (this follows from the fact that $(b_n)_ninmathbb N$ is a monotonic and bounded sequence).
    $endgroup$
    – José Carlos Santos
    Mar 15 at 18:47











  • $begingroup$
    By the way, is it necessary for $b_n$ to be monotonic? cause I haven't used that fact, I just used the boundedness.
    $endgroup$
    – shota kobakhidze
    Mar 15 at 23:16










  • $begingroup$
    See my previous comment. Without that sequence being monotonic, how do you know that the series $sum_n=1^inftylvert b_n-b_n+1rvert$ converges?
    $endgroup$
    – José Carlos Santos
    Mar 15 at 23:33










  • $begingroup$
    yes, I see. It was my bad. Thanks :)
    $endgroup$
    – shota kobakhidze
    Mar 16 at 0:00















$begingroup$
yeah, understood. Thanks for the response. By the way, saying that "we can make it as small as we want" is incorrect right? It would be more correct to say "bounded". Then the whole sum becomes "bounded" and not "as small as I wanted".
$endgroup$
– shota kobakhidze
Mar 15 at 18:38




$begingroup$
yeah, understood. Thanks for the response. By the way, saying that "we can make it as small as we want" is incorrect right? It would be more correct to say "bounded". Then the whole sum becomes "bounded" and not "as small as I wanted".
$endgroup$
– shota kobakhidze
Mar 15 at 18:38












$begingroup$
Oops! You are right. What you shoud use at that point is that $(s_n)_ninmathbb N$ is a boounded sequence and that the series $sum_n=1^inftylvert b_n-b_n+1rvert$ converges (this follows from the fact that $(b_n)_ninmathbb N$ is a monotonic and bounded sequence).
$endgroup$
– José Carlos Santos
Mar 15 at 18:47





$begingroup$
Oops! You are right. What you shoud use at that point is that $(s_n)_ninmathbb N$ is a boounded sequence and that the series $sum_n=1^inftylvert b_n-b_n+1rvert$ converges (this follows from the fact that $(b_n)_ninmathbb N$ is a monotonic and bounded sequence).
$endgroup$
– José Carlos Santos
Mar 15 at 18:47













$begingroup$
By the way, is it necessary for $b_n$ to be monotonic? cause I haven't used that fact, I just used the boundedness.
$endgroup$
– shota kobakhidze
Mar 15 at 23:16




$begingroup$
By the way, is it necessary for $b_n$ to be monotonic? cause I haven't used that fact, I just used the boundedness.
$endgroup$
– shota kobakhidze
Mar 15 at 23:16












$begingroup$
See my previous comment. Without that sequence being monotonic, how do you know that the series $sum_n=1^inftylvert b_n-b_n+1rvert$ converges?
$endgroup$
– José Carlos Santos
Mar 15 at 23:33




$begingroup$
See my previous comment. Without that sequence being monotonic, how do you know that the series $sum_n=1^inftylvert b_n-b_n+1rvert$ converges?
$endgroup$
– José Carlos Santos
Mar 15 at 23:33












$begingroup$
yes, I see. It was my bad. Thanks :)
$endgroup$
– shota kobakhidze
Mar 16 at 0:00




$begingroup$
yes, I see. It was my bad. Thanks :)
$endgroup$
– shota kobakhidze
Mar 16 at 0:00

















draft saved

draft discarded
















































Thanks for contributing an answer to Mathematics Stack Exchange!


  • Please be sure to answer the question. Provide details and share your research!

But avoid


  • Asking for help, clarification, or responding to other answers.

  • Making statements based on opinion; back them up with references or personal experience.

Use MathJax to format equations. MathJax reference.


To learn more, see our tips on writing great answers.




draft saved


draft discarded














StackExchange.ready(
function ()
StackExchange.openid.initPostLogin('.new-post-login', 'https%3a%2f%2fmath.stackexchange.com%2fquestions%2f3149615%2frudin-exercise-3-8-solution%23new-answer', 'question_page');

);

Post as a guest















Required, but never shown





















































Required, but never shown














Required, but never shown












Required, but never shown







Required, but never shown

































Required, but never shown














Required, but never shown












Required, but never shown







Required, but never shown







Popular posts from this blog

Lowndes Grove History Architecture References Navigation menu32°48′6″N 79°57′58″W / 32.80167°N 79.96611°W / 32.80167; -79.9661132°48′6″N 79°57′58″W / 32.80167°N 79.96611°W / 32.80167; -79.9661178002500"National Register Information System"Historic houses of South Carolina"Lowndes Grove""+32° 48' 6.00", −79° 57' 58.00""Lowndes Grove, Charleston County (260 St. Margaret St., Charleston)""Lowndes Grove"The Charleston ExpositionIt Happened in South Carolina"Lowndes Grove (House), Saint Margaret Street & Sixth Avenue, Charleston, Charleston County, SC(Photographs)"Plantations of the Carolina Low Countrye

random experiment with two different functions on unit interval Announcing the arrival of Valued Associate #679: Cesar Manara Planned maintenance scheduled April 23, 2019 at 00:00UTC (8:00pm US/Eastern)Random variable and probability space notionsRandom Walk with EdgesFinding functions where the increase over a random interval is Poisson distributedNumber of days until dayCan an observed event in fact be of zero probability?Unit random processmodels of coins and uniform distributionHow to get the number of successes given $n$ trials , probability $P$ and a random variable $X$Absorbing Markov chain in a computer. Is “almost every” turned into always convergence in computer executions?Stopped random walk is not uniformly integrable

How should I support this large drywall patch? Planned maintenance scheduled April 23, 2019 at 00:00UTC (8:00pm US/Eastern) Announcing the arrival of Valued Associate #679: Cesar Manara Unicorn Meta Zoo #1: Why another podcast?How do I cover large gaps in drywall?How do I keep drywall around a patch from crumbling?Can I glue a second layer of drywall?How to patch long strip on drywall?Large drywall patch: how to avoid bulging seams?Drywall Mesh Patch vs. Bulge? To remove or not to remove?How to fix this drywall job?Prep drywall before backsplashWhat's the best way to fix this horrible drywall patch job?Drywall patching using 3M Patch Plus Primer